Examen de pratique Flashcards

1
Q

Une femme de 18 ans consulte pour un examen de dépistage. Elle a eu 4 partenaires dans la dernière année. Elle dit avoir des saignements post-coïtaux à l’occasion. Le col est friable et saigne au contact. Il n’y a pas de leucorrhée anormale. Parmi les affirmations suivantes concernant l’infection à chlamydia trachomatis, laquelle est vraie?

A. Les gens âgés de 15-24 ans représentent le plus haut risque d’infection à chlamydia trachomatis.
B. Comme chez les femmes, l’infection extra-génitale à chlamydia trachomatis est anecdotique chez les hommes.
C. L’infection à chlamydia trachomatis se présente majoritairement par une leucorrhée et douleur pelvienne chez
les femmes.
D. On doit faire un prélèvement au col et pas ailleurs pour faire le diagnostic de chlamydia trachomatis.

A

A

How well did you know this?
1
Not at all
2
3
4
5
Perfectly
1
Q

Une femme de 21 ans consulte à l’urgence pour douleur pelvienne et à l’hypochondre droit. Elle a eu 3 partenaires depuis 6 mois. Elle se plaint aussi de leucorrhée verdâtre. Vous diagnostiquez une infection à gonocoques car un examen ayant été prélevé il y a 3 jours par un collègue est au dossier. Quelle affirmation est fausse?

A. Le diagnostic de la gonorrhée se fait uniquement par culture sur milieu charbon.
B. La cause de sa douleur à l’hypochondre droit s’explique probablement par des adhérences péri-hépatiques.
(Fitz-Hugh-curtis).
C. Vous devrez traiter les partenaires des 2 derniers mois.
D. Le traitement d’une cervicite à gonocoques NON COMPLIQUÉE consiste en une injection de Ceftriaxone 250
mg IM pour une dose.

A

A

How well did you know this?
1
Not at all
2
3
4
5
Perfectly
2
Q

Une femme de 53 ans consulte pour pertes vaginales incommodantes. Sans connaître les caractéristiques de ses leucorrhées, si le test de pH révèle qu’il est inférieur à 4,5, quel sera le diagnostic parmi les suivants?

A. Infection à trichomonas vaginalis
B. Vaginose bactérienne
C. Vaginite atrophique
D. Infection à candida albicans

A

D

How well did you know this?
1
Not at all
2
3
4
5
Perfectly
3
Q

Une femme de 32 ans consulte pour prurit vulvaire. L’examen révèle des lésions de grattage à la vulve et une leucorrhée blanche épaisse à l’allure de fromage cottage. Laquelle des affirmations suivantes est vraie?

A. Cette infection est une ITSS.
B. Un examen au microscope après ajout de KOH révèlera une odeur de poisson.
C. Un examen au microscope après ajout de KOH révèlera des filaments pseudo-mycéliens et des levures.
D. Le traitement consiste en un antibiotique de type Metronidazole intra-vaginal pour 7 jours.

A

C

How well did you know this?
1
Not at all
2
3
4
5
Perfectly
4
Q

Une femme de 30 ans vous consulte pour un examen annuel et vous demande quelle phase du cycle menstruel est généralement invariable en durée. Son but étant de déterminer le moment de son ovulation.

A. Phase folliculaire
B. Phase lutéale
C. Phase proliférative
D. Phase menstruelle

A

B

How well did you know this?
1
Not at all
2
3
4
5
Perfectly
5
Q

Une femme de 52 ans vous consulte pour son examen annuel. Elle n’a pas été menstruée depuis 13 mois, elle a quelques chaleurs peu incommodantes. Elle est seule et n’a pas d’activité sexuelle depuis 3 ans. Elle s’intéresse au maintien de sa santé et aimerait des conseils pour éviter l’ostéoporose. Laquelle des affirmations suivantes est vraie?

A. L’activité physique avec mise en charge est recommandée pour maintenir la santé osseuse.
B. Un supplément de calcium de 1200 mg par jour est recommandé même si elle en consomme dans son
alimentation.
C. Consommer des aliments riches en vitamine D est suffisant pour recevoir la dose de vitamine D recommandée
en ménopause.
D. Le tabagisme est favorable pour la transformation du cholécalciférol en vitamine D.

A

A

How well did you know this?
1
Not at all
2
3
4
5
Perfectly
6
Q

Une femme de 54 ans, ménopausée depuis 2 ans, souffrant de bouffées de chaleur nocturne, s’inquiète du cancer du sein si elle prenait des hormones de remplacement. Laquelle des affirmations suivantes est vraie?

A. C’est le supplément estrogénique seul qui augmente le risque de cancer du sein.
B. Le risque de cancer du sein est augmenté chez les utilisatrices d’estrogènes et progestatifs dans la même
proportion qu’il y ait antécédents familiaux de cancer du sein ou non.
C. Un antécédent de cancer du sein chez sa cousine maternelle est une contre-indication absolue à
l’hormonothérapie de remplacement chez elle.
D. L’utilisation d’estro-progestatif par voie transdermique est moins risquée pour le cancer du sein.

A

B

How well did you know this?
1
Not at all
2
3
4
5
Perfectly
7
Q

Une femme de 42 ans consulte avec son mari qui nous raconte qu’elle est vraiment insupportable avec de l’agressivité et des variations de l’humeur tellement graves qu’elle a perdu son emploi. Elle a des cycles réguliers aux 30 jours, ne prend aucune médication, ne consomme ni alcool ni drogues. Il a remarqué que ses sautes d’humeur disparaissent avec le début des menstruations à chaque mois. Quel examen vous aidera à prouver votre diagnostic de trouble dysphorique prémenstruel?

A. Prolactine
B. TSH
C. FSH-LH
D. Aucun examen sanguin

A

D

How well did you know this?
1
Not at all
2
3
4
5
Perfectly
8
Q

Une femme de 38 ans consulte pour irritabilité, tristesse et agressivité qui nuisent à sa vie de famille au point de se chicaner avec son mari et ses enfants. Cela perturbe aussi ses relations au travail. Vous soupçonnez un syndrome
prémenstruel. Laquelle des affirmations suivantes est vraie ?

A. Pour faire le diagnostic, on prescrit un contraceptif contenant de la Drospirénone qui fait disparaître ses symptômes.
B. On mesure la prolactinémie et la TSH. On s’assure qu’elle ne consomme pas de drogues ou médicaments.
C. On lui demande de faire un calendrier de ses symptômes qui doivent survenir en phase folliculaire
D. On lui demande de faire un calendrier de ses symptômes qui doivent survenir en phase lutéale

A

D

How well did you know this?
1
Not at all
2
3
4
5
Perfectly
9
Q

Une femme de 23 ans consulte pour aménorrhée depuis 12 mois. Le bilan sanguin hormonal de base est normal incluant FSH, TSH Prolactine et BHCG négatif. Vous faites aussi un test au progestatif qui est positif (la patiente saigne au retrait du progestatif). Quel est le diagnostic expliquant l’aménorrhée de cette patiente?

A. Hyperprolactinémie
B. Insuffisance ovarienne précoce
C. Anovulation chronique
D. Grossesse

A

C

How well did you know this?
1
Not at all
2
3
4
5
Perfectly
10
Q

Une jeune femme de 15 ans, 3 mois, consulte pour problèmes menstruels. Elle dit qu’elle n’est pas comme ses amies car elle n’a jamais saigné au niveau vaginal. Pourtant elle a des seins et des poils axillaires et pubiens. Quelle affirmation est vraie ?

A. On ne peut pas parler d’aménorrhée primaire quand les caractères sexuels sont présents.
B. On ne peut pas parler d’aménorrhée primaire avant 16 ans lorsque les caractères sexuels secondaires sont
présents
C. Elle aura ses menstruations sous peu car les caractères sexuels secondaires sont présents
D. Elle se qualifie pour un diagnostic d’aménorrhée primaire

A

D

How well did you know this?
1
Not at all
2
3
4
5
Perfectly
11
Q

Une femme de 34 ans vous appelle au bureau pour vous demander conseil car elle a oublié de changer son anneau contraceptif Nuvaring comme elle le fait d’habitude après 21 jours. Elle le porte depuis maintenant 28 jours et elle s’inquiète beaucoup d’être enceinte. Quelle est la réponse la plus juste ?

A. Elle doit retirer l’anneau immédiatement, prendre deux comprimés de Plan B et faire un test de grossesse
B. Elle peut garder l’anneau jusqu’à 35 jours sans s’inquiéter de l’efficacité contraceptive à condition d’en débuter
un nouveau immédiatement
C. Elle doit retirer l’anneau maintenant, faire un test de grossesse et si négatif, attendre 8 jours avant d’en
remettre un autre
D. Elle peut garder l’anneau jusqu’à 21 jours maximum sans s’inquiéter de l’efficacité contraceptive

A

B

How well did you know this?
1
Not at all
2
3
4
5
Perfectly
12
Q

Une femme de 31 ans vient d’accoucher de son premier enfant et désire l’allaiter. Elle vous demande quelles sont les conditions pour que son allaitement constitue un moyen de contraception optimal?

A. Accouchée moins de 12 mois, allaitement exclusif, aménorrhée
B. Accouchée moins de 6 mois, allaitement exclusif, aménorrhée
C. Accouchée moins de 6 mois, allaitement exclusif le jour, aménorrhée
D. Accouchée moins de 12 mois, allaitement exclusif le jour, oligoménorrhée

A

B

How well did you know this?
1
Not at all
2
3
4
5
Perfectly
13
Q

Concernant l’utilisation de spermicides en contraception, laquelle des affirmations suivantes est vraie?

A. Pour optimiser l’efficacité, il doit être appliqué entre 15 et 20 minutes avant la relation
B. Pour éviter l’irritation causée par le Nonoxynol-9, la femme doit se donner une douche vaginale en dedans de
8 heures de la relation.
C. Le Nonoxynol-9 est un agent chimique efficace pour diminuer le risque de transmission du VIH.
D. Les spermicides favorisent les infections vaginales à levures.

A

A

How well did you know this?
1
Not at all
2
3
4
5
Perfectly
14
Q

Femme de 32 ans, IMC 29, désir de grossesse d’ici 2-3 ans. Elle a fait une thrombophlébite du membre inférieur suite à une fracture en ski. Sa mère fait de l’ostéoporose à 67 ans. Elle veut une prescription de Depoprovera comme sa bonne amie qui est enchantée de ne plus avoir de menstruations. Parmi les affirmations suivantes, laquelle est vraie ?

A. Son antécédent de thrombophlébite du membre inférieur est une contre-indication au Depoprovera.
B. La fertilité après l’utilisation du Depo-Provera est compromise pour les 12 à 24 mois suivant l’arrêt.
C. Des saignements irréguliers et imprévisibles surviennent durant les 3 à 6 premiers mois d’utilisation.
D. Les femmes qui l’utilisent pendant plus d’un an souffriront d’ostéoporose avec un risque accru de fractures.

A

C

How well did you know this?
1
Not at all
2
3
4
5
Perfectly
15
Q

Qu’est-ce qui est responsable de la transformation de la gonade indifférenciée en testicule chez l’embryon masculin?

A. La production de TDF par le gène SRY
B. La sécrétion d’AMH par les cellules de Sertoli
C. La sécrétion de testostérone par les cellules de Leydig
D. La sécrétion de testostérone par les cellules de Sertoli

A

A

How well did you know this?
1
Not at all
2
3
4
5
Perfectly
16
Q

Le plancher pelvien est constitué entre autres des muscles releveurs de l’anus. Parmi les structures suivantes, laquelle fait partie des releveurs de l’anus ?

A. Muscle ischio-coccygien
B. Muscle pyriforme
C. Fascia pubo-cervical
D. Muscle sacro-spineux

A

A

17
Q

Femme 25 ans avec dysménorrhée invalidante l’empêchant de suivre ses cours chaque mois. Pas active sexuellement actuellement et aucun facteur de risque ITSS. En dehors des menstruations, elle fonctionne normalement sans douleur significative. Prend AINS depuis l’adolescence mais ce traitement ne suffit plus. Vit de la détresse psychologique secondaire à l’exacerbation de sa douleur, mais a un réseau de support actuellement. Echographie pelvienne normale. Examen pelvien normal. Quel traitement serait approprié pour l’aider?

A. Stérilet de cuivre
B. Contraceptifs oraux combinés
C. Opiacés
D. Psychothérapie

A

B

18
Q

Laquelle des affirmations suivante est fausse concernant l’atteinte inflammatoire pelvienne?

A. Nous devons réévaluer la réponse clinique 48-72h suite à l’initiation de l’antibiothérapie pour une Atteinte Inflammatoire Pelvienne.
B. La présence d’un abcès tubo-ovarien représente un critère d’hospitalisation.
C. La médication peut-être modifiée/cessée si les tests de PCR gonorrhée et chlamydia sont négatifs lors de la
visite de contrôle à 48h et qu’une amélioration clinique est notée.
D. La médication pour une atteinte inflammatoire pelvienne avec chlamydia positif est d’une durée de 14 jours,
comparativement au traitement pour une cervicite à chlamydia seul qui est d’une durée de 7 jours ou un traitement unidose selon les cas

A

C

19
Q

Femme 32 ans sans antécédent pertinent présente de la dysménorrhée progressive et de la dyspareunie depuis plusieurs semaines. Utilise le condom car a des migraines avec aura depuis quelques mois et a dû cesser son contraceptif oral. Partenaire stable depuis 4 ans. Elle n’est pas enceinte. À l’examen physique quel élément pourrait vous orienter vers un diagnostique d’endométriose?

A. Au toucher vaginal la présence de nodularités au site des ligaments utéro-sacrés avec douleur importante.
B. Au toucher bimanuel, utérus mobile et un peu sensible à la mobilisation.
C. Au spéculum la présence d’un col érythémateux avec écoulement muco-purulent.
D. Du péritonisme à la palpation de la fosse iliaque droite (ressaut+)

A

A

20
Q

Un nanisme fœtal présentant un index cardio-thoracique de plus de 50% a été diagnostiqué chez une femme primigeste présentement enceinte de 26 4/7 semaines. Après discussion, cette patiente opte pour une interruption volontaire de grossesse étant donné le létal pronostic fœtal. Quelle méthode d’induction du travail parmi les suivantes est recommandée chez cette patiente?

A. Amniotomie suivi d’oxytocine IV à 40 unités / 500 mL de salin physiologique à 80 mL / hre.
B. Mise en place de 2 à 3 tiges laminaires intracervicales puis misoprostol 200 mcg PV aux 4 heures.
C. Misoprostol 200 mcg PV aux 4 heures.
D. Misoprostol 200 mcg PV aux 4 heures et oxytocine IV à 40 unités / 500 mL de salin physiologique à 80 mL /
hre.

A

C

21
Q

Une femme enceinte de 7 2/7 semaines désire une interruption volontaire de grossesse par médication. Parmi les choix suivants, lequel est une contre-indication à l’utilisation de mifépristone et de misoprostol afin d’induire un avortement?

A. Un âge gestationnel de plus de 7 0/7 semaines.
B. La présence d’insuffisance surrénalienne chronique chez cette patiente.
C. La présence d’insuffisance rénale chronique chez cette patiente.
D. L’opposition du conjoint de la patiente à cette procédure (l’interruption volontaire de grossesse).

A

B

22
Q

Vous rencontrez Mme B, 39 ans, qui consulte pour l’apparition d’une nouvelle masse qu’elle palpe au niveau de son sein gauche. Elle a déjà eu des ponctions de kystes mammaires dans le passé. À l’examen vous palpez une masse bien définie, ronde et mobile de 3x3 cm. Celle-ci est douloureuse à la palpation. Vous tentez une ponction de mais malheureusement vous ne retirez aucun liquide.Quelle est votre conduite?

A. Vous la rassurez en lui disant qu’il s’agit sûrement d’un nouveau kyste et qu’il devrait disparaitre éventuellement.
B. Vous la référez rapidement en chirurgie pour une exérèse de cette lésion.
C. Vous lui prescrivez une mammographie gauche pour la rassurer.
D. Vous lui prescrivez une mammographie bilatérale et une échographie du sein gauche, avec possibilité de
ponction sous échographie.

A

D

23
Q

Vous êtes le nouveau médecin de famille de Mme L, 68 ans. Elle a subi une mastectomie partielle droite pour cancer du sein T1N0MO en 2014. Vous la rencontrez pour la première fois. Elle vous demande de passer des examens complets pour s’assurer qu’elle n’a pas de métastases, car sa voisine vient d’avoir un diagnostic de cancer du sein métastatique et Mme L est très inquiète pour elle-même. Par ailleurs sa dernière mammographie remonte à environ 3 ans. Au questionnaire, la revue des systèmes est négative et l’examen physique est normal. Quelle est votre conduite?

A. Vous demandez une consultation en oncologie pour évaluation et opinion.
B. Vous la rassurez et lui dites qu’il n’y a aucune indication de faire une investigation, autre qu’une
mammographie annuelle.
C. Vous prescrivez le bilan radiologique requis, soit une tomodensitométrie et une scintigraphie osseuse.
D. Vous lui prescrivez d’abord des marqueurs tumoraux et si ceux-ci s’avèrent anormaux, vous prescrirez un bilan
radiologique.

A

B

24
Q

Femme 68 ans se présente pour des plaintes urinaires basses. Elle a l’impression que sa vessie ne fonctionne plus. Elle urine tout le temps et peine à se rendre aux toilettes lorsque l‘envie lui prend. Elle doit effectuer une pression vaginale afin de bien vider sa vessie. L’inconfort est constant et gêne ses sorties. Vous suspectez un prolapsus antérieur du plancher pelvien. Comment un diagnostic de prolapsus pelvien est-il posé chez cette patiente?

A. Grâce à un examen pelvien en utilisant un système de stadification tel que le POP-Q.
B. Grâce une investigation radiologique complémentaire afin de confirmer le diagnostic.
C. Grâce à un bilan complémentaire incluant une analyse et culture d’urine.
D. L’anamnèse suffit dans ces circonstances.

A

A

25
Q

Femme 85 ans consulte avec sa fille. Maladie coronarienne stable et trouble cognitif modéré. Habite en résidence pour personne semi-autonome. Se plaint de difficulté à initier la miction. Rapporte devoir s’allonger dans son lit plus de 30 minutes avant de pouvoir faire une miction normale. Constipation chronique qu’elle traite avec des fibres solubles (Métamucil). Elle n’a pas eu d’infection urinaire depuis plusieurs années. Sa fonction rénale est stable. IMC de 36. Trouble à la marche. Vous notez un prolapsus antérieur de stade 3 et un prolapsus utérin de stade 2. Quel est selon vous la meilleure option de traitement adaptée à la patiente pour améliorer sa condition ?

A. Une chirurgie oblitérante (fermeture du vagin) puisqu’elle n’est plus active sexuellement.
B. Une sonde urinaire à demeure.
C. Un pessaire avec surveillance des résidus post-mictionnels après correction du prolapsus.
D. De la physiothérapie et la perte de poids.

A

C

26
Q

Laquelle des patientes suivantes est la plus à risque d’hyperplaise endométriale?

A. 33 ans, IMC 43, hirsutisme, ménométrorragies anémiantes depuis 6 mois
B. 60 ans, saignements post-coïtaux et dyspareunie superficielle
C. 50 ans, microrragies sous Mirena
D. 44 ans, infertilité anovulatoire avec syndrome des ovaires polykystiques, obésité et diabète de type 2 sous
Metformin

A

D

27
Q

Concernant les saignements utérins anormaux associés aux fibromes utérins, laquelle des affirmations est vraie?

A. Les progestatifs seuls ou combinés aux oestrogènes peuvent traiter les ménorragies.
B. Les anti-inflammatoires sont à éviter.
C. La myomectomie abdominale est le meilleur traitement des ménorragies associées à un fibrome de 8 cm sous-
séreux pédiculé.
D. L’hystérectomie est le traitement à privilégier chez les femmes qui ne souhaitent plus d’enfant.

A

C

28
Q

Juliette et Roméo débuteront sous peu un traitement de fécondation In vitro. Leurs médecins leur a bien expliqué les risques du traitement et ils sont particulièrement inquiets du risque de jumeaux avec les traitements de fertilité. Quel est le risque d’obtenir une grossesse gémellaire en fécondation in-vitro avec un transfert d’embryon unique?

A. 0
B. 1
C. 4
D. 10

A

B

29
Q

Une jeune femme de 21 ans vous consulte comme médecin de famille car elle a récemment eu un diagnostic de cancer du sein. Son oncologue a planifié une mastectomie qui sera suivi de chimiothérapie. L’oncologue l’a avisée que la chimiothérapie pouvait être toxique pour ses ovaires et pouvait entraîner une ménopause précoce. Que lui recommandez- vous?

A. Vous la rassurer en lui disant qu’il est peu probable d’être ménopausée après des traitements de chimiothérapie à son âge.
B. Vous l’aviser que si une ménopause précoce survient, vous lui prescrirez une hormonothérapie de remplacement pour protéger sa santé osseuse.
C. Vous la référer pour une consultation auprès d’un spécialiste en endocrinologie de la reproduction pour discuter les options de préservation de la fertilité comme la congélation d’ovules.
D. Vous l’avisez qu’une éventuelle ménopause précoce n’aurait aucun impact sur sa fertilité.

A

C

30
Q

Une femme de 55 ans, sous hormonothérapie de remplacement pour des symptômes vasomoteurs, présente des saignements de la post-ménopause. Elle s’administre deux (2) vaporisations quotidiennes d’un gel oestrogénique et prend un comprimé de 2,5 mg d’un progestatif de type acétate de médroxyprogestérone. Elle subit une biopsie de l’endomètre qui révèle insuffisant pour diagnostic mais sans évidence de malignité. Six (6) mois plus tard, les saignements se reproduisent. Quelle est LA MEILLEURE conduite à tenir dans cette situation?

A. Rassurer la patiente puisque la biopsie précédente ne révélait rien d’anormal.
B. Répéter la biopsie de l’endomètre.
C. Diminuer la dose de gel oestrogénique à une vaporisation quotidienne.
D. Augmenter la dose de progestatif.

A

B

31
Q

Laquelle des présentations cliniques suivantes est LA MOINS PROBABLE du cancer de l’endomètre?

A. Augmentation du volume abdominal et ascite
B. Douleur crampiforme pelvienne
C. Leucorrhée nauséabonde
D. Saignement post-coïtal à 64 ans

A

A

32
Q

Parmi les modalités de dépistage des cancers gynécologiques, quelle affirmation est VRAIE?

A. La cytologie cervicale (le test Pap) sert au dépistage du cancer de l’endomètre.
B. La recherche du VPH sur le col de l’utérus chez les femmes de moins de 30 ans est une bonne modalité de
dépistage du cancer du col utérin.
C. L’échographie en post-ménopause est une bonne modalité de dépistage du cancer de l’ovaire.
D. Une femme qui n’est pas ménopausée devrait subir une biopsie de l’endomètre pour des saignements utérins
anormaux persistants si le traitement de première ligne échoue ou si elle présente un risque de cancer de l’endomètre.

A

D

33
Q

Une femme de 44 ans se présente à la clinique sans rendez-vous pour un problème de lésions vulvaires. Vous portez un diagnostic de condylomes vulvaires. Elle n’est pas immunisée contre le VPH et elle vous pose des questions sur l’intérêt de se faire vacciner dans sa condition. Laquelle des affirmations suivantes est VRAIE?

A. La vaccination permettra de limiter les risques d’une nouvelle infection au VPH mais n’aura pas d’incidence sur sa condition actuelle.
B. La vaccination avec le vaccin nona-valent contre le VPH accélèrera la guérison des verrues actuelles.
C. La vaccination avec le vaccin nona-valent limitera les chances de transmissions des verrues génitales que la
patiente présente actuellement.
D. Si son conjoint est vacciné contre le VPH, il est totalement immunisé et ne contractera pas de condylomes.

A

A

34
Q

Une femme de 36 ans, fumeuse, est traitée pour un cancer du col de l’utérus. Elle prend du méthotrexate pour une arthrite rhumatoïde. Elle a eu plus de 20 partenaires sexuels dans sa vie. Elle n’a eu qu’une seule cytologie cervicale dans les 15 dernières années. Elle veut savoir pourquoi elle a le cancer du col de l’utérus. Quel est le facteur étiologique du cancer du col chez cette patiente?

A. Le tabagisme actif.
B. Le nombre élevé de partenaires sexuels.
C. L’immunosuppresion induite par le méthotrexate.
D. L’infection par le virus du papillome humain.

A

D

35
Q

Madame Leblanc a 28 ans et voit son médecin de famille dans le contexte d’un suivi longitudinal de sa santé. Elle n’a aucun problème de santé et est en couple stable avec sa partenaire depuis 3 ans. Elle a reçu la vaccination pour le VPH à l’école primaire. Sa mère a eu un cancer du col et elle s’inquiète de l’avoir elle aussi vu l’histoire familiale. Quel type de dépistage du cancer du col de l’utérus devrait lui être offert?

A. Aucun dépistage puisque ces femmes sont vaccinées.
B. Dépistage du VPH sur le col de l’utérus aux 5 ans.
C. Cytologie cervico-vaginale (test pap) aux 2 à 3 ans.
D. Cytologie cervico-vaginale (test pap) annuel vu l’histoire familiale.

A

C

36
Q

Une femme de 37 ans présente un prurit vulvaire incessant. À l’examen, sa vulve est rouge, sèche avec petites squames et lichénification. On note également des lésions d’excoriation (lésions de grattage). Elle rapporte aussi des lésions semblables depuis l’enfance dans le pli des coudes et sur les mains et les doigts. Quel est le diagnostic LE PLUS PROBABLE dans sa situation?

A. Lichen scléreux atrophique
B. Psoriasis vulvaire
C. Vulvite irritative
D. Eczéma vulvaire

A

D

37
Q

Une femme de 45 ans présente des lésions sur les grandes et les petites lèvres apparues après une relation sexuelle non protégée il y a 4 semaines de cela. Elles ont un aspect en crête de coq et ne sont pas prurigineuses. Lequel de ces traitements est LE MOINS approprié?

A. Traitement au LASER
B. Acide trichloro acétique
C. Exérèse chirurgicale
D. Expectative car les chances de résolution spontanée sont élevées après quelques mois

A

D

38
Q

Une femme de 18 ans se présente à l’urgence pour une douleur abdominale aigue suite à un rapport sexuel. Elle utilise le condom en guise de contraceptif. Elle ne fait pas de température. L’examen de son abdomen témoigne d’une douleur exquise en fosse iliaque droite avec un ressaut et une défense volontaire importante. Une masse pelvienne droite est palpable à l’examen gynécologique. Son bHCG est négatif. Son hémoglobine est à 120 g/L (normal 120-140 g/L) et il n’y a pas de leucocytose. L’échographie témoigne d’une masse ovarienne droite de 8 cm avec un peu de liquide libre autour et absence de flot au doppler, l’utérus est normal de même que l’appendice. Laquelle des conditions urgentes est LA PLUS probable chez cette femme.

A. Torsion ovarienne
B. Abcès tubo-ovarien
C. Kyste ovarien hémorragique
D. Grossesse ectopique

A

A

39
Q

Une femme de 44 ans se présente à l’urgence pour une douleur abdominale persistante depuis 2 mois. Elle a noté une augmentation du volume de son abdomen. L’échographie témoigne de masses ovariennes bilatérales (8 et 10 cm respectivement). Il y a également une large quantité d’ascite et des implants de carcinomatose qui touchent les anses intestinales et le péritoine. Parmi les diagnostics suivants, lequel est LE MOINS probable?

A. Cancer de l’ovaire séreux-papillaire
B. Endométriose sévère
C. Cancer de l’estomac avec métastases ovariennes
D. Tumeur séreuse borderline de l’ovaire avec implants péritonéaux

A

B